Difference between revisions of "2011 AIME I Problems/Problem 15"

(Solution)
(Solution 1)
(30 intermediate revisions by 19 users not shown)
Line 2: Line 2:
 
For some integer <math>m</math>, the polynomial <math>x^3 - 2011x + m</math> has the three integer roots <math>a</math>, <math>b</math>, and <math>c</math>.  Find <math>|a| + |b| + |c|</math>.
 
For some integer <math>m</math>, the polynomial <math>x^3 - 2011x + m</math> has the three integer roots <math>a</math>, <math>b</math>, and <math>c</math>.  Find <math>|a| + |b| + |c|</math>.
  
==Solution==
+
==Solution 1==
  
With Vieta's formula, we know that <math>a+b+c = 0</math>, and <math>ab+bc+ac = -2011</math>.  
+
From Vieta's formulas, we know that <math>a+b+c = 0</math>, and <math>ab+bc+ac = -2011</math>. Thus <math>a = -(b+c)</math>. All three of <math>a</math>, <math>b</math>, and <math>c</math> are non-zero: say, if <math>a=0</math>, then <math>b=-c=\pm\sqrt{2011}</math> (which is not an integer). <math>\textsc{wlog}</math>, let <math>|a| \ge |b| \ge |c|</math>. If <math>a > 0</math>, then <math>b,c < 0</math> and if <math>a < 0</math>, then <math>b,c > 0</math>. We have <cmath>-2011=ab+bc+ac = a(b+c)+bc = -a^2+bc</cmath>
 +
Thus <math>a^2 = 2011 + bc</math>. We know that <math>b</math>, <math>c</math> have the same sign. So <math>|a| \ge 45 = \lceil \sqrt{2011} \rceil</math>.  
  
 
+
Also, if we fix <math>a</math>, <math>b+c</math> is fixed, so <math>bc</math> is maximized when <math>b = c</math> . Hence, <cmath>2011 = a^2 - bc > \tfrac{3}{4}a^2 \qquad \Longrightarrow \qquad a ^2 < \tfrac{4}{3}\cdot 2011 = 2681+\tfrac{1}{3}</cmath>
<br />
+
So <math>|a| \le 51</math>. Thus we have bounded <math>a</math> as <math>45\le |a| \le 51</math>, i.e. <math>45\le |b+c| \le 51</math> since <math>a=-(b+c)</math>. Let's analyze <math>bc=(b+c)^2-2011</math>. Here is a table:  
<math>a,b,c\neq 0</math> since any one being zero will make the the other 2 <math>\pm \sqrt{2011}</math>.
 
 
 
<math>a = -(b+c)</math>. WLOG, let <math>|a| \ge |b| \ge |c|</math>.
 
 
 
Then if <math>a > 0</math>, then <math>b,c < 0</math> and if <math>a < 0</math>, <math>b,c > 0</math>.
 
 
 
<br />
 
<math>ab+bc+ac = -2011 = a(b+c)+bc = -a^2+bc</math>
 
 
 
<br />
 
<math>a^2 = 2011 + bc</math>
 
 
 
We know that <math>b</math>, <math>c</math> have the same sign. So <math>|a| \ge 45</math>. (<math>44^2<2011</math> and <math>45^2 = 2025</math>)
 
 
 
Also, <math>bc</math> maximize when <math>b = c</math> if we fixed <math>a</math>. Hence, <math>2011 = a^2 - bc > \frac{3}{4}a^2</math>.
 
 
 
So <math>a ^2 < \frac{(4)2011}{3} = 2681+\frac{1}{3}</math>.  
 
 
 
<math>52^2 = 2704</math> so <math>|a| \le 51</math>.
 
 
 
 
 
<br />
 
Now we have limited a to <math>45\le |a| \le 51</math>.
 
 
 
Let's us analyze <math>a^2 = 2011 + bc</math>.
 
 
 
<br />
 
 
 
Here is a table:
 
  
 
<table border = 1 cellspacing = 0 cellpadding = 5 style = "text-align:center;">
 
<table border = 1 cellspacing = 0 cellpadding = 5 style = "text-align:center;">
  
<tr><th><math>|a|</math></th><th><math>a^2 = 2011 + bc</math></th></tr>
+
<tr><th><math>|a|</math></th><th><math>bc=2011-a^2</math></th></tr>
  
 
<tr><td><math>45</math></td><td><math>14</math></td></tr>
 
<tr><td><math>45</math></td><td><math>14</math></td></tr>
<tr><td><math>46</math></td><td><math>14  + 91 =105</math></td></tr>
+
<tr><td><math>46</math></td><td><math>105</math></td></tr>
<tr><td><math>47</math></td><td><math>105 + 93 = 198</math></td></tr>
+
<tr><td><math>47</math></td><td><math>198</math></td></tr>
<tr><td><math>48</math></td><td><math>198 + 95 = 293</math></td></tr>
+
<tr><td><math>48</math></td><td><math>293</math></td></tr>
<tr><td><math>49</math></td><td><math>293 + 97 = 390</math></td></tr>
+
<tr><td><math>49</math></td><td><math>390</math></td></tr>
 
</table>
 
</table>
 
<br />
 
<br />
Line 54: Line 26:
 
<math>105 = (3)(5)(7)</math>, So <math>46</math> won't work. <math>198/47 > 4</math>,  
 
<math>105 = (3)(5)(7)</math>, So <math>46</math> won't work. <math>198/47 > 4</math>,  
  
<math>198</math> is not divisible by <math>5</math>, <math>198/6 = 33</math>, which is too small to get <math>47</math>
+
<math>198</math> is not divisible by <math>5</math>, <math>198/6 = 33</math>, which is too small to get <math>47</math>.
  
<math>293/48 > 6</math>, <math>293</math> is not divisible by <math>7</math> or <math>8</math> or <math>9</math>, we can clearly tell that <math>10</math> is too much
+
<math>293/48 > 6</math>, <math>293</math> is not divisible by <math>7</math> or <math>8</math> or <math>9</math>, we can clearly tell that <math>10</math> is too much.
  
  
Line 98: Line 70:
 
<br/>
 
<br/>
  
<math>|-49|+10+39 = \boxed{098}</math>
+
<math>|-49|+10+39 = \boxed{098}</math>.
 +
 
 +
 
 +
== Solution 3 ==
 +
Let us first note the obvious that is derived from Vieta's formulas: <math>a+b+c=0, ab+bc+ac=-2011</math>. Now, due to the first equation, let us say that <math>a+b=-c</math>, meaning that <math>a,b>0</math> and <math>c<0</math>. Now, since both <math>a</math> and <math>b</math> are greater than 0, their absolute values are both equal to <math>a</math> and <math>b</math>, respectively. Since <math>c</math> is less than 0, it equals <math>-a-b</math>. Therefore, <math>|c|=|-a-b|=a+b</math>, meaning <math>|a|+|b|+|c|=2(a+b)</math>. We now apply Newton's sums to get that <math>a^2+b^2+ab=2011</math>,or <math>(a+b)^2-ab=2011</math>. Solving, we find that <math>49^2-390</math> satisfies this, meaning <math>a+b=49</math>, so <math>2(a+b)=\boxed{098}</math>.
 +
 
 +
==Solution 4==
 +
We have
 +
 
 +
<math>(x-a)\cdot (x-b)\cdot (x-c)=x^3-(a+b+c)x+(ab+ac+bc)x-abc</math>
 +
 
 +
As a result, we have
 +
 
 +
<math>a+b+c=0</math>
 +
 
 +
<math>ab+bc+ac=-2011</math>
 +
 
 +
<math>abc=-m</math>
 +
 
 +
So, <math>a=-b-c</math>
 +
 
 +
As a result, <math>ab+bc+ac=(-b-c)b+(-b-c)c+bc=-b^2-c^2-bc=-2011</math>
 +
 
 +
Solve <math>b=\frac {-c+\sqrt{c^2-4(c^2-2011)}}{2}</math> and
 +
<math>\Delta =8044-3c^2=k^2</math>, where <math>k</math> is an integer
 +
 
 +
Cause <math>89<\sqrt{8044}<90</math>
 +
 
 +
So, after we tried for <math>2</math> times, we get <math>k=88</math> and <math>c=10</math>
 +
 
 +
then <math>b=39</math>, <math>a=-b-c=-49</math>
 +
 
 +
As a result, <math>|a|+|b|+|c|=10+39+49=\boxed{098}</math>
 +
 
 +
==Solution 5 (mod to help bash)==
 +
First, derive the equations <math>a=-b-c</math> and <math>ab+bc+ca=-2011\implies b^2+bc+c^2=2011</math>. Since the product is negative, <math>a</math> is negative, and <math>b</math> and <math>c</math> positive. Now, a simple mod 3 testing of all cases shows that <math>b\equiv \{1,2\} \pmod{3}</math>, and <math>c</math> has the repective value. We can choose <math>b</math> not congruent to 0, make sure you see why. Now, we bash on values of <math>b</math>, testing the quadratic function to see if <math>c</math> is positive. You can also use a delta argument like solution 4, but this is simpler. We get that for <math>b=10</math>, <math>c=39, -49</math>. Choosing <math>c</math> positive we get <math>a=-49</math>, so <math>|a|+|b|+|c|=10+29+39=\boxed{098}</math>
 +
~firebolt360
 +
 
 +
==Video Solution==
 +
 
 +
https://www.youtube.com/watch?v=QNbfAu5rdJI&t=26s ~ MathEx
  
 
== See also ==
 
== See also ==
  
{{AIME box|year=2011|num-b=14|after=-|n=I}}
+
{{AIME box|year=2011|num-b=14|after=Last Problem|n=I}}
 +
 
  
 
[[Category:Intermediate Algebra Problems]]
 
[[Category:Intermediate Algebra Problems]]
 
{{MAA Notice}}
 
{{MAA Notice}}

Revision as of 02:31, 27 January 2022

Problem

For some integer $m$, the polynomial $x^3 - 2011x + m$ has the three integer roots $a$, $b$, and $c$. Find $|a| + |b| + |c|$.

Solution 1

From Vieta's formulas, we know that $a+b+c = 0$, and $ab+bc+ac = -2011$. Thus $a = -(b+c)$. All three of $a$, $b$, and $c$ are non-zero: say, if $a=0$, then $b=-c=\pm\sqrt{2011}$ (which is not an integer). $\textsc{wlog}$, let $|a| \ge |b| \ge |c|$. If $a > 0$, then $b,c < 0$ and if $a < 0$, then $b,c > 0$. We have \[-2011=ab+bc+ac = a(b+c)+bc = -a^2+bc\] Thus $a^2 = 2011 + bc$. We know that $b$, $c$ have the same sign. So $|a| \ge 45 = \lceil \sqrt{2011} \rceil$.

Also, if we fix $a$, $b+c$ is fixed, so $bc$ is maximized when $b = c$ . Hence, \[2011 = a^2 - bc > \tfrac{3}{4}a^2 \qquad \Longrightarrow \qquad a ^2 < \tfrac{4}{3}\cdot 2011 = 2681+\tfrac{1}{3}\] So $|a| \le 51$. Thus we have bounded $a$ as $45\le |a| \le 51$, i.e. $45\le |b+c| \le 51$ since $a=-(b+c)$. Let's analyze $bc=(b+c)^2-2011$. Here is a table:

$|a|$$bc=2011-a^2$
$45$$14$
$46$$105$
$47$$198$
$48$$293$
$49$$390$


We can tell we don't need to bother with $45$,

$105 = (3)(5)(7)$, So $46$ won't work. $198/47 > 4$,

$198$ is not divisible by $5$, $198/6 = 33$, which is too small to get $47$.

$293/48 > 6$, $293$ is not divisible by $7$ or $8$ or $9$, we can clearly tell that $10$ is too much.


Hence, $|a| = 49$, $a^2 -2011 = 390$. $b = 39$, $c = 10$.

Answer: $\boxed{098}$

Solution 2

Starting off like the previous solution, we know that $a + b + c = 0$, and $ab + bc + ac = -2011$.

Therefore, $c = -b-a$.

Substituting, $ab + b(-b-a) + a(-b-a) = ab-b^2-ab-ab-a^2 = -2011$.

Factoring the perfect square, we get: $ab-(b+a)^2=-2011$ or $(b+a)^2-ab=2011$.

Therefore, a sum ($a+b$) squared minus a product ($ab$) gives $2011$..


We can guess and check different $a+b$’s starting with $45$ since $44^2 < 2011$.

$45^2 = 2025$ therefore $ab = 2025-2011 = 14$.

Since no factors of $14$ can sum to $45$ ($1+14$ being the largest sum), a + b cannot equal $45$.

$46^2 = 2116$ making $ab = 105 = 3 * 5 * 7$.

$5 * 7 + 3 < 46$ and $3 * 5 * 7 > 46$ so $46$ cannot work either.


We can continue to do this until we reach $49$.

$49^2 =  2401$ making $ab = 390 = 2 * 3 * 5* 13$.

$3 * 13 + 2* 5 = 49$, so one root is $10$ and another is $39$. The roots sum to zero, so the last root must be $-49$.


$|-49|+10+39 = \boxed{098}$.


Solution 3

Let us first note the obvious that is derived from Vieta's formulas: $a+b+c=0, ab+bc+ac=-2011$. Now, due to the first equation, let us say that $a+b=-c$, meaning that $a,b>0$ and $c<0$. Now, since both $a$ and $b$ are greater than 0, their absolute values are both equal to $a$ and $b$, respectively. Since $c$ is less than 0, it equals $-a-b$. Therefore, $|c|=|-a-b|=a+b$, meaning $|a|+|b|+|c|=2(a+b)$. We now apply Newton's sums to get that $a^2+b^2+ab=2011$,or $(a+b)^2-ab=2011$. Solving, we find that $49^2-390$ satisfies this, meaning $a+b=49$, so $2(a+b)=\boxed{098}$.

Solution 4

We have

$(x-a)\cdot (x-b)\cdot (x-c)=x^3-(a+b+c)x+(ab+ac+bc)x-abc$ 

As a result, we have

$a+b+c=0$

$ab+bc+ac=-2011$

$abc=-m$

So, $a=-b-c$

As a result, $ab+bc+ac=(-b-c)b+(-b-c)c+bc=-b^2-c^2-bc=-2011$

Solve $b=\frac {-c+\sqrt{c^2-4(c^2-2011)}}{2}$ and $\Delta =8044-3c^2=k^2$, where $k$ is an integer

Cause $89<\sqrt{8044}<90$

So, after we tried for $2$ times, we get $k=88$ and $c=10$

then $b=39$, $a=-b-c=-49$

As a result, $|a|+|b|+|c|=10+39+49=\boxed{098}$

Solution 5 (mod to help bash)

First, derive the equations $a=-b-c$ and $ab+bc+ca=-2011\implies b^2+bc+c^2=2011$. Since the product is negative, $a$ is negative, and $b$ and $c$ positive. Now, a simple mod 3 testing of all cases shows that $b\equiv \{1,2\} \pmod{3}$, and $c$ has the repective value. We can choose $b$ not congruent to 0, make sure you see why. Now, we bash on values of $b$, testing the quadratic function to see if $c$ is positive. You can also use a delta argument like solution 4, but this is simpler. We get that for $b=10$, $c=39, -49$. Choosing $c$ positive we get $a=-49$, so $|a|+|b|+|c|=10+29+39=\boxed{098}$ ~firebolt360

Video Solution

https://www.youtube.com/watch?v=QNbfAu5rdJI&t=26s ~ MathEx

See also

2011 AIME I (ProblemsAnswer KeyResources)
Preceded by
Problem 14
Followed by
Last Problem
1 2 3 4 5 6 7 8 9 10 11 12 13 14 15
All AIME Problems and Solutions

The problems on this page are copyrighted by the Mathematical Association of America's American Mathematics Competitions. AMC logo.png